Prove that det(A*) = [det(A)]*

  • Thread starter Thread starter jack476
  • Start date Start date
Click For Summary
The discussion focuses on proving that the determinant of the complex conjugate matrix A* is equal to the complex conjugate of the determinant of A. The initial approach involves using mathematical induction, starting with a base case for 2x2 matrices and extending the argument to n x n matrices. The inductive step utilizes properties of determinants and the distributive nature of complex conjugation. Feedback suggests that a direct approach might be more efficient, emphasizing the properties of sums and products in relation to determinants. The conclusion reinforces that the proof is valid for all n x n matrices with complex elements.
jack476
Messages
327
Reaction score
125

Homework Statement


Prove that the determinant of A*, the matrix of the complex conjugates of the elements of A, is equal to the complex conjugate of the determinant.

Homework Equations


None, but the hint provided with the problem suggested using an induction argument.

The Attempt at a Solution


To establish the base case, I show that:
##
\begin{vmatrix}
a & b\\
c & d
\end{vmatrix}
^*=(ad-cb)^*=a^*d^*-c^*b^* =
\begin{vmatrix}
a^* & b^*\\
c^* & d^*
\end{vmatrix}
##

For my inductive hypothesis, I claim that for the square matrix Ann, that det(Ann*) = det(Ann)*

Then for the inductive step, I use the fact that for any matrix Mnn, det(Mn+1,n+1) = αdet(Mnn) + β, where α is some constant and β is the sum of all of the other terms in that determinant (that is, I use the fact that the determinant of Mnn is a factor in the equation for the determinant of Mn+1, n+1), along with the fact that complex conjugation is distributive under complex addition and multiplication, so that:

det(An+1, n+1)* = [αdet(Ann) + β]* = α*det(Ann)* + β* = det(An+1, n+1*)

So that the inductive step is completed, and therefore for all nxn matrices of complex elements, the determinant of the complex conjugate matrix is the complex conjugate of that matrix's determinant.

Any feedback would be greatly appreciated.
 
Physics news on Phys.org
jack476 said:

Homework Statement


Prove that the determinant of A*, the matrix of the complex conjugates of the elements of A, is equal to the complex conjugate of the determinant.

Homework Equations


None, but the hint provided with the problem suggested using an induction argument.

The Attempt at a Solution


To establish the base case, I show that:
##
\begin{vmatrix}
a & b\\
c & d
\end{vmatrix}
^*=(ad-cb)^*=a^*d^*-c^*b^* =
\begin{vmatrix}
a^* & b^*\\
c^* & d^*
\end{vmatrix}
##
I would establish the basis for ##n = 1##, so ##1 \times 1## matrices.
jack476 said:
For my inductive hypothesis, I claim that for the square matrix Ann, that det(Ann*) = det(Ann)*

Then for the inductive step, I use the fact that for any matrix Mnn, det(Mn+1,n+1) = αdet(Mnn) + β, where α is some constant and β is the sum of all of the other terms in that determinant (that is, I use the fact that the determinant of Mnn is a factor in the equation for the determinant of Mn+1, n+1), along with the fact that complex conjugation is distributive under complex addition and multiplication, so that:

det(An+1, n+1)* = [αdet(Ann) + β]* = α*det(Ann)* + β* = det(An+1, n+1*)

So that the inductive step is completed, and therefore for all nxn matrices of complex elements, the determinant of the complex conjugate matrix is the complex conjugate of that matrix's determinant.

Any feedback would be greatly appreciated.
I understand the idea, and it is close. However, I prefer to be a bit more explicit. Why don't you write ##\text{det}\,{M_{n+1,n+1}}## as a cofactor expansion along, say, the first row, so
$$
\text{det}\,{M_{n+1,n+1}} = \sum_{k=1}^{n + 1}{(-1)^{1 + k}P_{1,k}}
$$
where ##P_{1,k}## is the ##(1,k)##-minor (an ##n \times n## determinant) and hence ##(-1)^{1 + k}P_{1,k}## is the corresponding cofactor.
 
Last edited:
jack476 said:

Homework Statement


Prove that the determinant of A*, the matrix of the complex conjugates of the elements of A, is equal to the complex conjugate of the determinant.

Homework Equations


None, but the hint provided with the problem suggested using an induction argument.

The Attempt at a Solution


To establish the base case, I show that:
##
\begin{vmatrix}
a & b\\
c & d
\end{vmatrix}
^*=(ad-cb)^*=a^*d^*-c^*b^* =
\begin{vmatrix}
a^* & b^*\\
c^* & d^*
\end{vmatrix}
##

For my inductive hypothesis, I claim that for the square matrix Ann, that det(Ann*) = det(Ann)*

Then for the inductive step, I use the fact that for any matrix Mnn, det(Mn+1,n+1) = αdet(Mnn) + β, where α is some constant and β is the sum of all of the other terms in that determinant (that is, I use the fact that the determinant of Mnn is a factor in the equation for the determinant of Mn+1, n+1), along with the fact that complex conjugation is distributive under complex addition and multiplication, so that:

det(An+1, n+1)* = [αdet(Ann) + β]* = α*det(Ann)* + β* = det(An+1, n+1*)

So that the inductive step is completed, and therefore for all nxn matrices of complex elements, the determinant of the complex conjugate matrix is the complex conjugate of that matrix's determinant.

Any feedback would be greatly appreciated.

As Krylov has pointed out, you are very nearly done. However, wouldn't a direct, non-indiction approach be faster? All we need to know are three things: (1) the conjugate of a sum is the sum of the conjugates; (2) the conjugate of a product is the product of the conjugates; and (3) one of the definitions of ##\det(A)## is:
\det(A) = \sum_{i_1, i_2,\ldots, i_n} \epsilon(i_1,i_2, \ldots, i_n) a_{1 i_1} a_{2 i_2} \cdots a_{n i_n},
where
\epsilon(i_1,i_2, \ldots, i_n) = \begin{cases} +1, &amp; i_1, i_2, \ldots i_n \; \text{is an even permutation of } \: 1,2, \ldots, n \\<br /> -1, &amp; i_1, i_2, \ldots i_n \; \text{is an odd permutation of } \: 1,2, \ldots, n \\<br /> 0, &amp; \text{otherwise}<br /> \end{cases}<br />
 
Question: A clock's minute hand has length 4 and its hour hand has length 3. What is the distance between the tips at the moment when it is increasing most rapidly?(Putnam Exam Question) Answer: Making assumption that both the hands moves at constant angular velocities, the answer is ## \sqrt{7} .## But don't you think this assumption is somewhat doubtful and wrong?

Similar threads

  • · Replies 4 ·
Replies
4
Views
2K
Replies
17
Views
2K
  • · Replies 25 ·
Replies
25
Views
3K
  • · Replies 11 ·
Replies
11
Views
2K
  • · Replies 5 ·
Replies
5
Views
5K
  • · Replies 17 ·
Replies
17
Views
2K
Replies
2
Views
2K
Replies
4
Views
2K
Replies
4
Views
7K
  • · Replies 6 ·
Replies
6
Views
2K